Which statement is true regarding the graphed functions?

Which Statement Is True Regarding The Graphed Functions?

Answers

Answer 1

Answer:

f(2) = 0 and g(2) = 0

f(2) = g(2) TRUE

b) f(0) = 2 and g(0) = -2

f(0) = g(0) FALSE

c) f(2) = 0 and g(0) = -2

f(2) = g(0) FALSE

d) f(0) = 2 and g(2) = 0

f(0) = g(2) FALSE


Related Questions

Viết các lũy thừa sau dưới dạng một lũy thừa hoặc các tích các lũy thừa có số nguyên và số nguyên A.8 mũ 2 B. 654​

Answers

Step-by-step explanation:

câu trả lời là trong hình ảnh trên

can I get a walkthrough on these problems please?

Answers

Answer:

Step-by-step explanation:

52 + 27 = angle 1 (sum of two interior opposite angle is equal to the exterior angle formed)

79 = angle 1

angle 2 + 71 = 92 degree (sum of two interior opposite angle is equal to the exterior angle formed)

angle 2 = 92 - 71

angle 2 = 21 degree

Algebra
write down the coefficient of x in 3(x)4x
A 12
B 3
C 0


by Emmanuel okorie​

Answers

Answer:

nah its a.

Step-by-step explanation:

bakit?(#boyinthebrainly)

Use the number line to find the measure of SV

Answers

Answer:

8

Step-by-step explanation:

S = -7 to V= 1 you count from -7 to 1

Yulet rolls 2 fair dice and adds the results from each.
Work out the probability of getting a total that is a factor of 24.

Answers

2,4,6,8,10,12,14,16,18,20,22,24!.. so 2 x 12= (24).....

an angle is 21 degree more than twice its complement find it
QUICK
ASAP now before tomorrow Please

Answers

Answer: 67°

Step-by-step explanation:

Complementary angle refers to the angle that when added together will be equal to 90°.

Let the measurement of the angle's complement be x.

Therefore, the angles are x and 2x + 21. Since it's a complementary angle, therefore,

x + 2x + 21 = 90

3x + 21 = 90

3x = 90 - 21

3x = 69

x = 69°/3

x = 23°

Therefore, the angle will be:

= 2x + 21

= 2(23°) + 21°

= 46° + 21°

= 67°

The angle is 67°.

(x+1)^2 . (x^2+1) = 0

Answers

Answer:

[tex]x^4+2x^3+2x^2+2x+1[/tex]

Step-by-step explanation:

Given that,

[tex](x+1)^2 . (x^2+1) = 0[/tex]

We know that, [tex](a+b)^2=a^2+b^2+2ab[/tex]

So,

[tex](x+1)^2=x^2+1+2x[/tex]

So,

[tex](x+1)^2 . (x^2+1) = (x^2+1+2x)(x^2+1)\\\\=x^2\times x^2+x^2+2x^3+x^2+1+2x\\\\=x^4+2x^3+2x^2+2x+1[/tex]

So, the value of the given expression is equal to[tex]x^4+2x^3+2x^2+2x+1[/tex]

helpppp please!!!!!!

Answers

Answer:

x-8

Step-by-step explanation:

since we dont know how much gas is in the tank, lets make that X.

and since we subtract 8 gallons from how much is in the tank, we make that X-8

Answer:

Inequality:

x < 8

The Graph Would be the Hallow Dot on the 8,  then it would go the left way.

Perform the transformations

Answers

I can’t see the question it looks blurry

Cedarburg's zoo has two elephants. The male elephant weighs 3 3/4 tons and the female
elephant weighs 7/12 of a ton. How much more does the male weigh than the female?

Answers

Answer:

3.16666667 tons

Step-by-step explanation:

hope this helps have a good evening

Total health care costs in the United States in 2016 was $1.7 trillion. he US population was 290.9 million. What was the average amount spent per person on health care?

Answers

Answer:

$5,843.9

Step-by-step explanation:

2016

Total health care cost in US = $1.7 trillion

= $1,700,000,000,000

Population in US = 290.9 million.

= 290,900,000

Average amount spent per person on health care = Total health care cost in US / Population in US

= 1,700,000,000,000 / 290,900,000

= $5843.9326228944

Approximately,

$5,843.9

To the nearest whole number

= $5,844

PLEASE HELP
what is the amount of an investment $10,000 compounded quarterly for 3 years at a rate of 4 percent?

Answers

Answer:

10000 times 3 times 0.04 = 1200

Answer:

11268.25

Step-by-step explanation:

[tex]P(1+\frac{i}{n})^{n*t}=10000(1+\frac{.04}{4})^{4*3}=11268.2503[/tex]

Rewrite the following quadratic function in vertex form. Then, determine if it has a maximum or minimum and say what that value is.

y = -x 2 + 6x + 5



HELP PLEASE!!!!!!

Answers

Answer:

-(x-3)²+14

maximum

(3,14)

Step-by-step explanation:

y= -x²+6x+5

y-5= -x²+6x

y-5= -(x²-6x)

complete the square

y-14= -(x²-6x+9)

x²-6x-9= -(x-3)²

y-14= -(x-3)²

y= -(x-3)²+14

maximum because a is negative

vertex is (3,14)

Does this help? Vertex form: y(x) = 4x + 5

It's a quartile question.

Answers

Answer:

D. 8

Step-by-step explanation:

the upper quartile = 8

simplify

x - ( - 6 x )​

Answers

Answer:

7x

Step-by-step explanation:

Answer:

answer

x - ( - 6x )

x + 6x

= 7x

Question 1
Which of the following numbers is greater than 6 and less than 8?
60
O 780
70

Answers

Answer:

The number which is more than 6 and less than 8 is 7.

Step-by-step explanation:

There are the natural numbers.

To find the number which is more than 6 but less than 8.

The numbers which are more than 6 are 7, 8, 9, 10, ....

The numbers which are less than 8 are ...,3, 4, 5, 6, 7

So, the number which is more than 6 and less than 8 is 7.

the cost for renting a car is $29.00 a day plus $0.29 for every kilometre after the first 100 km. what is the cost of renting a car after 14 days and 320 km?
(linear equations)

Answers

Answer:

$469.80

Step-by-step explanation:

First, let's find out how many fees would be paid exclusively for renting it out for 14 days.

29.00 * 14 = $406

They have to pay 406 dollars just for having the car for 14 days.

Now let's find out how much will need to paid for driving for 320 km.

We can first take away 100 km because the fee only starts afterwards.

320 - 100 = 220

220 * 0.29 = 63.8

63.80 + 406 = 469.80

change from improper fraction to mix number 51/7​

Answers

Answer:

7 3/7

Step-by-step explanation:

Each month, Kelsey donates 1/5 of her allowance to her school for supplies. 1/2 of that amount goes to the chorus class. How much of her allowance goes to supplies for the chorus class?
Please show your work.
Thank you :)

Answers

every month 1/10 of her allowance goes to chores class because 1/5x1/2 = 1/10 or 1/2 of 1/5 is 1/10

Solve for x.
3:12 = x:16
64
4
2 1/4

Answers

Hi!

i think We can set up a proportion to solve this

\frac{3}{12} \frac{x}{16}

12

3

16

x

Cross multiply

3 x 16 = 48

48/12 = 4

The answer is 4

Hope this helps! :)

PLSSS HELPPP no linkss plsss im begging

Answers

Answer:

Step-by-step explanation:

(First correct answer for brainliest) Find the length of JM

Answers

Answer:

JM = 18 units

Step-by-step explanation:

∆MHN and ∆JKH are similar (by A.A.A axiom)

so, their sides are proportional

i.e.

(JM+18)/(20+15) = 18/15

or, JM + 18 = 540/15

or, JM = 36-18

hence, JM = 18

Find the area of the circle. (Picture attached). Thank you!!

Answers

the area is 254.5 hope it helps

Answer:

254.5cm²

Step-by-step explanation:

I assumed that the units is cm.

hope it helps!!!

A cyclist travels 8 km from home to the park. She leaves home at 10:15 and arrives at the park at 10:30. What is her average speed in km/h?

Answers

Answer:

speed = 32km/hr

Step-by-step explanation:

Time

10:15 to 10:30 = 1/4 hr

speed = distance/time

speed = 8 / (1/4)

speed = 32km/hr

What is the weight of one boxe of a 165 ounces

Answers

Answer:

10 pounds

Step-by-step explanation:

165 ounces = 10 pounds

Answer:

10 pounds, 5 ounces

Step-by-step explanation:

The phrasing of the question is unclear. 165 ounces is itself a measure of weight, but it can be further broken down into pounds and ounces.

Recall that 1 pound = 16 ounces. Therefore, 165 ounces = 10 pounds with remainder of 5 ounces. Or in simple decimals it is 165 ounces / 16 ounces = 10.3125 pounds.

Shirts-2-Go sells t-shirts for a base price of $12 per shirt plus a fixed fee of $3 shipping and handling for the whole order. Shirts PLUS sells t-shirts for a base price of $8 per shirt plus a fixed fee of $23 shipping and handling for the whole order. Let x represent the number of shirts ordered and let y represent the total cost of the order.

y = 12x + 3

y = 8x + 23

How many shirts would you have to purchase for the total cost to be the same at both companies?

Answers

Answer:

5 shirts

Step-by-step explanation:

Y=12x+3 equation 1

y=8x+23 equation 2

12x+3=8x+23 substituting y value will cause equations to equal

4x=20

x=5 shirts

check answer

y=12x+3

y=12(5)+3

y=60+3

y=63

y=8x+23

y=8(5)+23

y=40+23

y=63

Find the area of AABC. Round your answer to the nearest tenth.
mZA = 23º, mZC = 39, b = 14.6
a. 44.5
b. 29.7
C. 49.4
d. 77.1

Answers

Answer:

the answer is 77.1

Can somebody help I’ll give brainless plss

Answers

Answer:

$93.22

Step-by-step explanation:

bill = $79

tip = 18%

tip amount = tip % of bill

=18/100 * $79

=$1422/100

=$14.22

total cost = $79 + $14.22

=$93.22

Answer:

93.22

Step-by-step explanation:

Find the amount of the tip

79*18%

79 * .18

14.22

Add this to the amount of the bill

79+14.22

93.22


Dan, Harry and Regan sell cars.
Dan sells x cars.
Harry sells 5 more cars than Dan.
Regan sells twice as many cars as Dan.
Write an expression, in terms of x, for the mean number of cars Dan, Harry and Regan sell.

Answers

Answer: (4x + 5)/3

Explanation:

D = x
H = x + 5
R = 2x

Mean = (D + H + R)/3
= (x + x + 5 + 2x)/3
= (4x + 5)/2

Which similarity postulate or theorem can be used to verify that the two
triangles shown below are similar?

A. SAS theorem
B. SSS theorem
C. AA postulate
D. Similarity cannot be determined.

Answers

Answer:

C

Step-by-step explanation:

AA postulate because we are given 2 of the 3 angles of each triangle. They are both the same in each triangle. Also, we can find the 3rd angle in each triangle. Since both triangles have the exact same angles, they are similar.

The given triangles ΔABC and ΔLMN are similar by AA postulate.

What is a triangle?

A triangle is a three sided polygon that consist of three edges and three vertices .

Here two given triangles are ΔABC and ΔLMN. In these two triangles ∠A=∠L=53° and ∠B=∠M= 72°.

Therefore, the two corresponding angles of triangles ΔABC and ΔLMN are equal . So the third angle of ΔABC is also equal to the corresponding angle of ΔLMN.

Hence two triangles are similar by AA postulate.

Thus two triangles are similar by AA postulate. So option (C) is correct.

To learn more about triagles click here:

https://brainly.com/question/24147586

#SPJ7

Other Questions
Ingrid drew a diagram to compare two ways in which biodiversity can be protected.Which label belongs in the area marked Y?has the goal of releasing animals in the wildis often done in nature preservesusually protects entire ecosystemsfocuses on increasing the population of a single species Evaluate the function at the given values of x. Round to 4 decimal places, if necessary. =fx7x What do you think is more important?School or Money?Both is not allowed. How did free enterprise help Oprah Winfrey in her business endeavors? what is the key code for a tamagotchi how does justice relate to fairness and equality? Translate the phrase into an algebraic expression.3 more than b Will mark brainliest PLEASE HEPWrite out a paragraph showing your understanding of scarcity what can be known about this situation? ASSIGNMENTSCOURSESAssignment - 1. Pre-TestAttempt 1 of 1SECTION 1 of 1QUESTICWorld War I marked the first war in which all of the following new technologies were used except foratomic bombsairplanestankspoison gas On a coordinate plane, triangle A B C has points (negative 2, negative 1), (negative 2, negative 3), (negative 5, negative 1). Triangle ABC is translated 6 units to the right and 1 units up. What are the coordinates of C'? C'(4, 2) C'(4, 0) C'(4, 2) C'(1, 2) evaluate g(x)=x/x-3, if g(1/2) Look up lamentings in the dictionary. What did Lennox hear in the night? What is the human female reproductive cycle regulated by? 1) What is the control group? 2) What is the experimental group? 3) What is the independent (manipulated) variable? 4) What is the dependent (responding) variable? 5) What should Tina's conclusion be? Using the following image, complete the statement below. I got all the answers I need the ones that is blank A 21-year-old woman presents with a 3-month history of a black mole on her right calf. She tells you that the lesion is enlarging and expanding. It began to itch about 3 weeks ago, and it has bled 2 times. She thinks that there may have been a mole near the same spot previously, but she is not certain. Her general health is good; there is no history of chronic illness, hospitalizations, or surgeries. She works as a professional model for a large advertisement agency. She does not take any prescription medication; she does not use tobacco, alcohol, or recreational drugs. Although she has dark hair, she has a fair skin, and she says that she usually burns with even short sun exposure. She does occasionally use a tanning booth prior to modeling events and vacations. There is no family history of skin cancer.VS stable, she looks anxious, but she is otherwise well.There is a dark brown-black nodule on the right calf 1 cm in diameter.On the surface of the nodule, there is a tiny area of crusting. There are no hairs. The nodule is asymmetrical, and its border is sharply demarcated; the color is uniform, and the elevation is regular. There is a narrow (1-2 mm) rim of erythema around most of the nodule. She has a sprinkling (about 25-30 in all) of melanocytic nevi on her trunk and legs. There is no significant local or distal lymphadenopathy. The liver is not palpable. The remainder of the physical examination is unremarkable.What is the most likely diagnosis? Given the equation 4square root of x minus 3 = -12, solve for x and identify if it is an extraneous solution.A. x = 0, solution is extraneousB. x = 0, solution is not extraneousC. x = 12, solution is extraneousD. x = 12, solution is not extraneous Which expression is equivalent to 10k + 17 - 7j - 18 - 11k?-8jk - 1-7j - k - 1-7j + k + 1-8j - kPLEASE HURRY !!!!20 POINTS!!!